Proof using Permutation Symbols

Click For Summary
The discussion focuses on proving the vector identity using permutation symbols, specifically the equation (AxB)x(CxD)=(A.BxD)C-(A.BxC)D=(A.CxD)B-(B.CxD)A. Participants emphasize the tedious nature of the proof, which requires applying specific rules related to permutation tensors and the Kronecker delta. Key rules include the relationships between cross products and dot products expressed in terms of permutation symbols. The initial steps involve rewriting the identity in component form to facilitate the proof. Overall, the discussion highlights the complexity of manipulating permutation symbols in vector identities.
BlazNProdigy
Messages
1
Reaction score
0

Homework Statement


Proove that...
(AxB)x(CxD)=(A.BxD)C-(A.BxC)D=(A.CxD)B-(B.CxD)A
using Permutation Symbols

Homework Equations

The Attempt at a Solution


I am confused about what to do after the third line from 'vela's response' (Post #2 from the reference link below).

Reference https://www.physicsforums.com/threa...rmutation-tensor-and-kroenecker-delta.454568/
 
Physics news on Phys.org
The problem is straight-forward, just tedious. You just apply the following rules:

  1. (X \times Y)^c = \epsilon_{abc} X^a Y^b
  2. (X \cdot Y) = \delta_{ab} X^a Y^b
  3. \epsilon_{abc} = \epsilon_{bca} = \epsilon_{cab} = -\epsilon_{bac} = -\epsilon{acb} = - \epsilon_{cba}
  4. \epsilon_{abc} \epsilon_{ade} = \delta_{bd} \delta_{ce} - \delta_{be} \delta_{cd}
  5. \epsilon_{abc} \delta_{ae} = \epsilon_{ebc}
To get the ball rolling, rewrite what you're being asked to prove in terms of components:

((A \times B) \times (C \times D))^c = (A \cdot (B \times D)) C^c - (A \cdot (B \times C)) D^c = (A \cdot (C \times D)) B^c - (B \cdot (C \times D)) A^c
 
Last edited:
Question: A clock's minute hand has length 4 and its hour hand has length 3. What is the distance between the tips at the moment when it is increasing most rapidly?(Putnam Exam Question) Answer: Making assumption that both the hands moves at constant angular velocities, the answer is ## \sqrt{7} .## But don't you think this assumption is somewhat doubtful and wrong?

Similar threads

  • · Replies 3 ·
Replies
3
Views
1K
Replies
5
Views
17K
  • · Replies 15 ·
Replies
15
Views
5K
  • · Replies 8 ·
Replies
8
Views
5K
Replies
5
Views
2K
Replies
1
Views
2K
Replies
2
Views
6K
  • · Replies 2 ·
Replies
2
Views
2K
  • · Replies 4 ·
Replies
4
Views
4K
  • · Replies 8 ·
Replies
8
Views
3K